Browse Source

added images; added solution

Martin Thoma 12 years ago
parent
commit
0e005dabca

BIN
documents/GeoTopo/GeoTopo.pdf


+ 1 - 0
documents/GeoTopo/GeoTopo.tex

@@ -34,6 +34,7 @@
 \usepackage{tikz-3dplot}    % draw
 \usepackage{tkz-fct}        % draw
 \usetikzlibrary{3d,calc,intersections,er,arrows,positioning,shapes.misc,patterns,fadings,decorations.pathreplacing}
+\usepackage{tqft}
 \usepackage{cleveref} % has to be after hyperref, ntheorem, amsthm
 \usepackage{shortcuts}
 

+ 0 - 1
documents/GeoTopo/Kapitel1-UB.tex

@@ -19,7 +19,6 @@
     Zeigen Sie:
     \begin{enumerate}[label=(\alph*)]
         \item Jedes $U_{a,b}$ und jede einelementige Teilmenge von $\mdz$ ist abgeschlossen.
-        \item Die $U_{a,b}$ bilden eine Basis der Topologie.
         \item $\Set{-1, 1}$ ist nicht offen.
         \item Es gibt unendlich viele Primzahlen.
     \end{enumerate}

+ 4 - 2
documents/GeoTopo/Kapitel1.tex

@@ -298,7 +298,7 @@ Auch gibt es Mengen, die sowohl abgeschlossen als auch offen sind.
     $\Rightarrow$ Widerspruch   $\qed$
 \end{beweis}
 
-\section{Stetigkeit}
+\section{Stetigkeit}\index{Stetigkeit|(}
 \begin{definition} \xindex{stetig} \xindex{Homöomorphismus}
     Seien $X, Y$ topologische Räume und $f:X \rightarrow Y$ eine Abbildung.
 
@@ -470,6 +470,7 @@ sodass $\pi$ stetig wird.
 %%%%%%%%%%%%%%%%%%%%%%%%%%%%%%%%%%%%%%%%%%%%%%%%%%%%%%%%%%%%%%%%%%%%%
 % Mitschrieb vom 31.10.2013                                         %
 %%%%%%%%%%%%%%%%%%%%%%%%%%%%%%%%%%%%%%%%%%%%%%%%%%%%%%%%%%%%%%%%%%%%%
+\index{Stetigkeit|)}
 \section{Zusammenhang}\index{Zusammenhang|(}
 \begin{definition}\xindex{zusammenhängend}
     Ein Raum $X$ heißt \textbf{zusammenhängend}, wenn es keine offenen
@@ -799,7 +800,7 @@ $\qed$
 %%%%%%%%%%%%%%%%%%%%%%%%%%%%%%%%%%%%%%%%%%%%%%%%%%%%%%%%%%%%%%%%%%%%%
 % Mitschrieb vom 07.11.2013                                         %
 %%%%%%%%%%%%%%%%%%%%%%%%%%%%%%%%%%%%%%%%%%%%%%%%%%%%%%%%%%%%%%%%%%%%%
-\section{Wege und Knoten}
+\section{Wege und Knoten}\index{Knoten|(}
 \begin{definition}\xindex{Weg}\xindex{Weg!geschlossener}\xindex{Weg!einfacher}
     Sei $X$ ein topologischer Raum. 
     \begin{enumerate}[label=\alph*)]
@@ -1003,6 +1004,7 @@ $\qed$
     \label{fig:reidemeister-zuege}
     \caption{Ein 3-gefärber Kleeblattknoten}
 \end{figure}
+\index{Knoten|)}
 
 % Die Übungsaufgaben sollen ganz am Ende des Kapitels sein.
 \input{Kapitel1-UB}

+ 19 - 2
documents/GeoTopo/Kapitel2.tex

@@ -210,7 +210,7 @@ U_i = \Set{(x_0: \dots : x_n) \in \mdp^n(\mdr) | x_i \neq 0} &\rightarrow \mdr^n
     \end{enumerate}
 \end{beispiel}
 
-\begin{definition}\textbf{Mannigfaltigkeit!mit Rand}
+\begin{definition}\xindex{Mannigfaltigkeit!mit Rand}
     Sei $X$ ein Hausdorffraum mit abzählbarer Basis der Topologie.
     $X$ heißt $n$-dimensionale \textbf{Mannigfaltigkeit mit Rand},
     wenn es einen Atlas $(U_i, \varphi_i)$ gibt, wobei $U_i \subseteq X_i$
@@ -221,7 +221,24 @@ U_i = \Set{(x_0: \dots : x_n) \in \mdp^n(\mdr) | x_i \neq 0} &\rightarrow \mdr^n
 \end{definition}
 
 \begin{beispiel}
-    \todo[inline]{Viele Bilder: Pair of pants, sphere with a hole, halbraum...}
+    \begin{figure}[ht]
+        \centering
+        \subfloat[Halbraum]{
+            \input{figures/topology-halfspace.tex}
+            \label{fig:half-space}
+        }%
+
+        \subfloat[Pair of pants]{
+            \input{figures/topology-pair-of-pants.tex}
+            \label{fig:pair-of-pants}
+        }%
+        \subfloat[Sphäre mit einem Loch]{
+            \input{figures/topology-sphere-with-hole.tex}
+            \label{fig:sphere-with-hole}
+        }%
+        \label{Mannigfaltigkeiten mit Rand}
+        \caption{Beispiele für Mannigfaltigkeiten mit Rand}
+    \end{figure}
 \end{beispiel}
 
 \begin{definition}\xindex{Rand}

+ 32 - 1
documents/GeoTopo/Loesungen.tex

@@ -25,7 +25,38 @@
 \end{solution}
 
 \begin{solution}[\ref{ub1:aufg4}]
-    \todo[inline]{Lösung schreiben}
+    \textbf{Teilaufgabe a)} 
+
+    \textbf{Beh.:}  $\forall a \in \mdz: \Set{a}$ ist abgeschlossen.
+
+    Sei $a \in \mdz$ beliebig. Dann gilt:
+    \todo[inline]{Hat jemand diesen Beweis?}
+
+    \textbf{Teilaufgabe b)} 
+
+    \textbf{Beh.:} $\Set{-1, 1}$ ist nicht offen
+
+    \textbf{Bew.:} durch Widerspruch
+
+    Annahme: $\Set{-1, 1}$ ist offen.
+
+    Dann gibt es $T \subseteq \fB$, sodass $\bigcup_{M \in T} M = \Set{-1, 1}$.
+    Aber alle $U \in \fB$ haben unendlich viele Elemente. Auch endlich
+    viele Schnitte von Elementen in $\fB$ haben unendlich viele
+    Elemente $\Rightarrow$ keine endliche nicht-leere Menge kann
+    in dieser Topologie offen sein $\Rightarrow \Set{-1,1}$ ist
+    nicht offen. $\qed$
+
+    \textbf{Beh.:} Es gibt unendlich viele Primzahlen.
+
+    \textbf{Bew.:} durch Widerspruch
+
+    Annahme:  Es gibt nur endlich viele Primzahlen $p \in \mdp$
+
+    Dann ist 
+    \[\mdz \setminus \Set{-1, +1} \overset{\text{FS d. Arithmetik}}= \bigcup_{p \in \mdp} U_{0,p}\]
+    endlich. Das ist ein Widerspruch zu $|\mdz|$ ist unendlich und
+    $|\Set{-1,1}|$ ist endlich. $\qed$
 \end{solution}
 
 \begin{solution}[\ref{ub2:aufg4}]